Frage:
Überbestimmen die Maxwellschen Gleichungen die elektrischen und magnetischen Felder?
Warrick
2012-01-27 14:53:38 UTC
view on stackexchange narkive permalink

Maxwellsche Gleichungen geben zwei Vektor- und zwei skalare (Differential-) Gleichungen an. Das impliziert 8 Komponenten in den Gleichungen. Aber zwischen den Vektorfeldern $ \ vec {E} = (E_x, E_y, E_z) $ und $ \ vec {B} = (B_x, B_y, B_z) $ gibt es nur 6 Unbekannte. Wir haben also 8 Gleichungen für 6 Unbekannte. Warum ist das kein Problem?

Soweit ich weiß, lautet die Antwort im Grunde, weil die Gleichungen nicht wirklich unabhängig sind, aber ich habe nie eine klare Erklärung gefunden. Vielleicht ist die richtige Richtung in diesem Artikel auf arXiv.

Entschuldigung, wenn dies ein Repost ist. Ich habe einige Diskussionen in PhysicsForums gefunden, aber hier keine ähnliche Frage.

Sieben antworten:
Luboš Motl
2012-01-27 15:13:06 UTC
view on stackexchange narkive permalink

Es ist kein Problem, da zwei der acht Gleichungen Einschränkungen sind und nicht ganz unabhängig von den verbleibenden sechs.

Die Einschränkungsgleichungen sind die skalaren, $$ {\ rm div } \, \, \ vec D = \ rho, \ qquad {\ rm div} \, \, \ vec B = 0 $$ Stellen Sie sich $ \ vec D = \ epsilon_0 \ vec E $ und $ \ vec B = \ mu_0 vor \ vec H $ der Einfachheit halber überall.

Wenn diese Gleichungen im Ausgangszustand erfüllt sind, werden sie jederzeit sofort erfüllt. Dies liegt daran, dass die Zeitableitungen dieser nicht dynamischen Gleichungen ("nicht dynamisch" bedeutet, dass sie nicht dazu bestimmt sind, Zeitableitungen von Feldern selbst zu bestimmen; sie enthalten keine Zeitableitungen) aus den verbleibenden 6 Gleichungen berechnet werden können . Wenden Sie einfach $ {\ rm div} $ auf die verbleibenden 6 Komponentengleichungen an, $$ {\ rm curl} \, \, \ vec E + \ frac {\ partiell \ vec B} {\ partiell t} = 0, \ qquad { \ rm curl} \, \, \ vec H- \ frac {\ partiell \ vec D} {\ partiell t} = \ vec j. $$ Wenn Sie $ {\ rm div} $ anwenden, verschwinden die Curl-Begriffe, weil $ {\ rm div} \, \, {\ rm curl} \, \, \ vec V \ equiv 0 $ eine Identität ist und Sie erhalten $$ \ frac {\ partiell ({\ rm div} \, \, \ vec B)} {\ partiell t} = 0, \ qquad \ frac {\ partiell ({\ rm div} \, \, \ vec D. )} {\ partielle t} = - {\ rm div} \, \, \ vec j. $$ Die erste Gleichung impliziert, dass $ {\ rm div} \, \, \ vec B $ Null bleibt, wenn es im Anfangszustand Null wäre. Die zweite Gleichung kann unter Verwendung der Kontinuitätsgleichung für $ \ vec j $, $$ \ frac {\ partielle \ rho} {\ partielle t} + {\ rm div} \, \, \ vec j = 0 $$ ( dh wir gehen davon aus, dass dies für die Quellen gilt), um $$ \ frac {\ partiell ({\ rm div} \, \, \ vec D- \ rho)} {\ partiell t} = 0 $$, also $ {\ rm div} \, \, \ vec D- \ rho $ bleibt auch jederzeit Null, wenn es im Ausgangszustand Null ist.

Lassen Sie mich erwähnen, dass unter den 6 + 2-Komponenten-Maxwell-Gleichungen 4 von denen, die $ \ vec E, \ vec B $ betreffen, gelöst werden können, indem $ \ vec E, \ vec B $ in Form von vier geschrieben wird Komponenten $ \ Phi, \ vec A $. In dieser Sprache bleiben nur die verbleibenden 4 Maxwell-Gleichungen übrig. Wie oben gezeigt, sind jedoch jeweils nur 3 von ihnen wirklich unabhängig. Das ist auch in Ordnung, da die vier Komponenten von $ \ Phi, \ vec A $ nicht ganz bestimmt sind: Eine dieser Komponenten (oder eine Funktion) kann durch die 1-Parameter-Invarianz $ U (1) $ geändert werden. P. >

Was Lubos sagt, ist, dass Skalargleichungen als Konsequenzen von Vektorgleichungen, Ladungserhaltung und Anfangsbedingungen betrachtet werden können. Zum Beispiel ist $ div B = 0 $ die Folge der Erhaltung der magnetischen Ladung und des Fehlens magnetischer Ladungen zum Anfangszeitpunkt, da $ div B = const $ die Folge von $ curl E = - \ frac {\ partielles B} ist. {\ partielle t} $ und $ divB = 0 $ zum Anfangszeitpunkt.
Lubosh, $ \ vec {E}, \ vec {B} $ werden über 6 Zeit- und Raum * -Derivate * von $ \ phi $ und $ \ vec {A} $ ausgedrückt; Deshalb gibt es eine Mehrdeutigkeit der Potenziale.
Lieber Wladimir, ich habe Ihre Frage ausführlich beantwortet. Nochmal. Es gibt eine 1-Parameter-Mehrdeutigkeit in den 4 Potentialen - die U (1) -Gradinvarianz - weil lokal in der Raumzeit die 4 Potentiale nur durch 3 Gleichungen beschränkt sind, Curl H = $ j + \ partielles D / \ partielles t $. Die vierte Gleichung mit Strömen, $ {\ rm div} \, \, D = \ rho $, ist nicht unabhängig: Ihre zeitliche Ableitung folgt aus den vorherigen drei. Die verbleibenden 3 + 1-Gleichungen für $ B, E $ werden automatisch erfüllt, wenn $ B, E $ als 4 Potentiale ausgedrückt werden, es handelt sich um Bianchi-Identitäten.
Natürlich ist die Art und Weise, wie wir Potentiale einführen, nicht willkürlich, sondern spezifisch für die Maxwell-Gleichungen. Jede Spezifikation ist eine Einschränkung im Vergleich zur Willkür. Sagen Sie mir nun, wie viele unabhängige elektrische und magnetische Feldkomponenten gibt es in der Elektromagneto-Statik?
Sehr geehrte @Vladimir, in einem Moment, z. Im Ausgangszustand sind $ \ vec E, \ vec B $ an jedem Punkt unabhängig - also 6 Komponenten pro Punkt -, aber sie werden durch $ {\ rm div} \, \, \ vec D = \ rho $ und eingeschränkt $ {\ rm div} \, \, \ vec B = 0 $. Es sind also effektiv vier unabhängige Komponenten pro Punkt. Auch hier sind statische Felder nicht das gleiche Problem wie die dynamischen, die Zählung unterscheidet sich für generische Punkte vom Ausgangszustand.
@Lubosh: Sie sagen fast korrekte Dinge, aber schauen Sie, $ \ rm div \ vec {D} = \ rho $ ist keine Einschränkung für die Vektorkomponenten $ \ vec {D} $. Zum Beispiel sind zwei Polynome $ P_1 = a + bx $ und $ P_2 = c + bx $ linear unabhängig, aber ihre Ableitungen sind gleich. Eine Einschränkung für die Komponenten sieht aus wie $ \ vec {a} \ vec {D} = 1 $ oder so.
Ist Lubos oder Lubosh besser für die "Hutmacher" geeignet?
Sehr geehrte @Vladimir, $ {\ rm div} \, \ vec D = \ rho $ ist eine Einschränkung für $ \ vec D $, sicherlich im technischen Sinne. Es ist keine rein algebraische Einschränkung; Wenn es so wäre, wäre es lösbar und man könnte einfach einige Komponenten von $ \ vec D $ sofort löschen. Stattdessen enthält es räumliche Ableitungen. Dieser Unterschied erlaubt es jedoch nur, dass sich das gesamte $ \ vec D $ im Raum konstant bewegt: unabhängig vom Raum. An einzelnen Punkten der anfänglichen Schicht ist das Vorhandensein von Ableitungen für die Zählung irrelevant und es gibt 1 Einschränkung pro Punkt (mit Ausnahme eines Punkts im Raum), genau wie wenn es eine algebraische wäre
Lieber @Nick, Lubos oder Lubosh ist sicherlich einfacher zu schreiben und ich werde nicht beleidigt. Viele Menschen, auch diejenigen außerhalb Mittel- und Osteuropas, in denen š möglicherweise über die Tastatur eingegeben wird, können das Zeichen in etwa einer Sekunde durch Kopieren und Einfügen usw. schreiben. Es ist also kein großes Opfer, wenn sie es schreiben korrekt. Aber ja, š wird als sh ausgesprochen.
@LubošMotl Vielen Dank für eine klare und ausführliche Antwort.Soweit ich weiß, verwendet dieser Ansatz die Kontinuitätsgleichung für $ \ vec {j} $ als zusätzliches Axiom zur ** dynamischen ** Maxwell-Gleichung.Wenn ich richtig liege, können Sie bitte erläutern, warum ein solches zusätzliches Axiom als selbstverständlich gilt.Ich bin verwirrt, ob es sich um ein zusätzliches Axiom oder nur um eine Definition handelt.Ohne die Kontinuitätsgleichung zu verwenden, wie Sie gezeigt haben, würden wir $ \ partielle_t (\ nabla \ cdot \ vec {D}) = - \ nabla \ cdot \ vec {j} $ erhalten, aber nicht $ \ partielle_t (\ nabla \ cdot\ vec {D} - \ rho) = 0 $.Vielen Dank für Ihre Zeit!
Die Kontinuitätsgleichung für rho und j folgt aus allen Maxwellschen Gleichungen - einschließlich derjenigen ohne Zeitableitung.Wenn Sie versuchen, E, B zu finden, die Maxwells Gleichungen für rho, j lösen, die der Kontinuitätsgleichung nicht entsprechen, gibt es überhaupt keine Lösungen.Sie könnten Lösungen finden, wenn rho, j gegen die Kontinuitätsgleichung verstößt und Sie die zeitunabhängigen Maxwell-Gleichungen fallen lassen.Aber dieses Gleichungssystem, Annahmen wären nicht Lorentz-invariant, also ist es nicht besonders interessant.
@Lubos Motl - Entspricht nicht die Kontinuitätsgleichung $ \ frac {\ partiell \ rho} {\ partiell t} + {\ rm div} \ vec j = 0 $ ** ** aus der vorherigen Formel $ \ frac {\ partiell({\ rm div} \, \, \ vec D)} {\ teilweise t} = - {\ rm div} \ vec j $, wenn wir $ {\ rm div} \ vec D = \ rho $ in dieses setzenFormel?Ich spüre hier eine gewisse Zirkularität. Was nützt es, $ \ frac {\ partiell ({\ rm div} \ vec D- \ rho)} {\ partiell t} = 0 $ zu schreiben, wenn dies aus zwei gleichen Formeln stammt?Gibt es Fälle, in denen $ {\ rm div} \ vec D- \ rho $ ungleich Null ist?
Ja, die von Ihnen beschriebene Implikation gilt, aber es gibt nichts Zirkuläres.In der Physik betrachten wir nur Konfigurationen, in denen alle Maxwell-Gleichungen gelten, einschließlich div D = rho.Sie könnten diese Gleichung entfernen, aber die anderen drei Maxwell-Gleichungen würden immer noch implizieren, dass (div D - rho) zeitunabhängig ist, ansonsten aber jede Funktion von x, y, z.Es wäre kein schrecklich interessantes Gleichungssystem, Funktionen ungleich Null von x, y, z zuzulassen, und die Theorie wäre nicht Lorentz-invariant.
Der Punkt ist, dass Maxwells Gleichungen nicht ganz unabhängig voneinander sind - drei von ihnen können differenziert und kombiniert werden, um die Zeitableitung der vierten abzuleiten.Interessant ist jedoch nur das System, in dem all dieses scheinbar (nur durch Zählen der Bedingungen) "übervollständige" System auferlegt wird.
Qmechanic
2012-02-05 23:43:51 UTC
view on stackexchange narkive permalink

I) Lassen Sie uns zum Spaß die Frage von OP auf $ n $ Raumzeitdimensionen verallgemeinern und überprüfen, wie die Zählung von Gl. und Freiheitsgrade (d.o.f.) arbeiten in dieser allgemeinen Umgebung. Wir werden die Antwort von Lubos Motl als Vorlage für diesen Teil verwenden. Wir werden auch eine spezielle relativistische $ (-, +, \ ldots, +) $ -Notation mit $ c = 1 $ verwenden, wobei $ \ mu, \ nu \ in \ {0, \ ldots, n-1 \} $ bezeichnen Raumzeitindizes, während $ i, j \ in \ {1, \ ldots, n-1 \} $ räumliche Indizes bezeichnen. Maxwell-Gleichungen sind die folgenden.

  1. Quellenfreie Bianchi-Identitäten: $$ {\ rm d} F ~ = ~ 0 \ qquad \ qquad \ Linker rechter Pfeil \ qquad \ qquad \ sum _ {\ rm cycl. ~ \ Mu, \ nu, \ lambda} d _ {\ lambda} F _ {\ mu \ nu} ~ = ~ 0, \ qquad \ qquadF ~: = ~ \ frac { 1} {2} F _ {\ mu \ nu} ~ {\ rm d} x ^ {\ mu} \ wedge {\ rm d} x ^ {\ nu}. $$ Hier $$ \ left (\ begin {array } {c} n \ cr 3 \ end {array} \ right) {\ rm ~ Bianchi ~ identities} ~ = ~ \ left (\ begin {array} {c} n-1 \ cr 3 \ end {array} \ rechts) {\ rm ~ Einschränkungen} ~ + ~ \ links (\ begin {array} {c} n-1 \ cr 2 \ end {array} \ right) {\ rm ~ dynamical ~ eqs.} $$$$ ~ = ~ ({\ rm No ~ magnetische ~ Monopole ~ Gleichungen.}) ~ + ~ ({\ rm Faradays ~ Gesetz}). $$

  2. Maxwell-Gl. mit Quellbegriffen: $$ d _ {\ mu} F ^ {\ mu \ nu} ~ = ~ -j ^ {\ nu}. $$ Hier $$ n {\ rm ~ source ~ eqs.} ~ = ~ 1 { \ rm ~ Einschränkung} ~ + ~ (n-1) {\ rm ~ dynamische ~ Gleichungen.} $$$$ ~ = ~ ({\ rm Gauß '~ Gesetz}) ~ + ~ ({\ rm Ampere's ~ Gesetz ~ mit ~ Verschiebung ~ Term}). $$

  3. ol>

    Wir haben die Terminologie verwendet, dass eine dynamische Gleichung Zeitableitungen enthält, während eine Einschränkung nicht. Also die Anzahl der dynamischen Gl. ist

    $$ \ left (\ begin {array} {c} n-1 \ cr 2 \ end {array} \ right) ~ + ~ (n-1) ~ = ~ \ left (\ begin {array} {c} n \ cr 2 \ end {array} \ right), $$

    , was genau mit

    $$ {\ rm the ~ number ~} \ übereinstimmt links (\ begin {array} {c} n \ cr 2 \ end {array} \ right) {\ rm ~ von ~} F _ {\ mu \ nu} {\ rm ~ Felder} $$$$ ~ = ~ \ links (\ begin {array} {c} n-1 \ cr 2 \ end {array} \ right) {~ \ rm magnetische ~ Felder ~} F_ {ij} ~ + ~ (n-1) {\ rm ~ electric ~ Felder ~} F_ {i0}. $$

    Maxwell-Gl. mit Quelltermen implizieren die Kontinuitätsgleichung

    $$ d _ {\ nu} j ^ {\ nu} ~ = ~ -d _ {\ nu} d _ {\ mu} F ^ {\ mu \ nu} ~ = ~ 0, \ qquad \ qquad F ^ {\ mu \ nu} ~ = ~ -F ^ {\ nu \ mu}, $$

    Man muss also verlangen, dass die Hintergrundquellen $ j ^ {\ nu} $ der Kontinuitätsgleichung entsprechen.

    Aus Gründen der Konsistenz sollte die zeitliche Ableitung jeder der Einschränkungen verschwinden. Im Fall der nichtmagnetischen Monopolgleichungen folgt dies aus dem Faradayschen Gesetz. Im Fall des Gaußschen Gesetzes folgt dies aus dem modifizierten Ampere-Gesetz und der Kontinuitätsgleichung.

    II) Im vorherigen Abschnitt (I) wurde die Zählung in Bezug auf das Array $ \ left (\ begin {) durchgeführt } {c} n \ cr 2 \ end {array} \ right) $ Feldstärken $ F _ {\ mu \ nu} $. In Bezug auf die $ n $ Messpotentiale $ A _ {\ mu} $ läuft die Zählung wie folgt ab. Die Bianchi-Identitäten sind jetzt trivial erfüllt,

    $$ F ~ = ~ {\ rm d} A \ qquad \ qquad A ~: = ~ A _ {\ mu} ~ {\ rm d} x ^ { \ mu}. $$

    Es gibt immer noch die $ n $ Maxwell-Gleichungen. mit Quellbegriffen

    $$ (\ Box \ delta ^ {\ mu} _ {\ nu} -d ^ {\ mu} d _ {\ nu}) A ^ {\ nu} ~ = ~ - j ^ {\ mu}, \ qquad \ qquad \ Box ~: = ~ d _ {\ mu} d ^ {\ mu}. $$

    Es gibt eine einzelne Spurweite d.o.f. wegen der Eichensymmetrie $ A \ zu A + {\ rm d} \ Lambda $ und $ F \ zu F $. Wenn ein Messgerät-Fix unter Verwendung der Lorenz-Messgerät-Bedingung

    $$ d _ {\ mu} A ^ {\ mu} ~ = ~ 0, $ $

    die Maxwell-Gl. werden $ n $ entkoppelte Wellengleichungen

    $$ \ Box A ^ {\ mu} (x) ~ = ~ -j ^ {\ mu} (x). $$

    Durch eine räumliche Fourier-Transformation werden diese zu entkoppelten linearen ODEs zweiter Ordnung mit konstanten Koeffizienten

    $$ (d ^ 2_t + \ vec { k} ^ 2) \ hat {A} ^ {\ mu} (t; \ vec {k}) ~ = ~ \ hat {j} ^ {\ mu} (t; \ vec {k}), $$

    , das ab einem Anfangszeitpunkt $ t_0 $ für alle Zeiten $ t $ gelöst werden kann, vgl. OPs Frage. [Man sollte überprüfen, ob die Lösung

    $$ \ hat {A} ^ {\ mu} (t; \ vec {k}) ~ = ~ \ int {\ rm d} t ^ {\ prime } ~ G (tt ^ {\ prime}; \ vec {k}) ~ \ hat {j} ^ {\ mu} (t ^ {\ prime}; \ vec {k}), \ qquad \ qquad (d ^ 2_t + \ vec {k} ^ 2) G (tt ^ {\ prime}; \ vec {k}) ~ = ~ \ delta (tt ^ {\ prime}), $$

    erfüllt den Lorenz Messgerät Zustand. Dies folgt aus der Kontinuitätsgleichung.]

    III) Es ist interessant, die vollständige Lösung $ \ tilde {A} ^ {\ mu} (k) $ in $ k ^ {\ nu} $ - abzuleiten. Impulsraum ohne Messgerätfixierung. Die Fourier-transformierten Maxwell-Gl. Lesen Sie

    $$ M ^ {\ mu} {} _ {\ nu} ~ \ tilde {A} ^ {\ nu} (k) ~ = ~ \ tilde {j} ^ {\ mu} (k), \ qquad \ qquad M ^ {\ mu} {} _ {\ nu} ~: = ~ k ^ 2 \ delta ^ {\ mu} _ {\ nu} -k ^ {\ mu} k _ {\ nu}. $$

    Um fortzufahren, muss die Matrix $ M ^ {\ mu} {} _ {\ nu} $ auf festes $ k ^ {\ lambda} $ analysiert werden. Es gibt drei Fälle.

    1. Konstanter Modus $ k ^ {\ mu} = 0 $. Dann verschwindet die Matrix $ M ^ {\ mu} {} _ {\ nu} = 0 $ identisch. Maxwell Gl. können nur erfüllt werden, wenn $ \ tilde {j} ^ {\ mu} (k = 0) = 0 $ Null ist. Das Eichpotential $ \ tilde {A} _ {\ mu} (k = 0) $ wird durch Maxwell-Gleichungen überhaupt nicht eingeschränkt, dh es gibt eine vollständige $ n $ -Parameterlösung.

    2. Massiver Fall $ k ^ 2 \ neq 0 $. Die Matrix $ M ^ {\ mu} {} _ {\ nu} $ ist diagonalisierbar mit dem Eigenwert $ k ^ 2 $ (mit der Multiplizität $ n-1 $) und dem Eigenwert $ 0 $ (mit der Multiplizität $ 1 $). Letzteres entspricht einem reinen Messmodus $ \ tilde {A} ^ {\ mu} ~ \ propto ~ k ^ {\ mu} $. Die vollständige Lösung ist eine $ 1 $ -Parameterlösung der Form $$ \ tilde {A} ^ {\ mu} (k) ~ = ~ \ frac {\ tilde {j} ^ {\ mu} (k)} {k ^ 2} ~ + ~ ik ^ {\ mu} \ tilde {\ Lambda} (k). $$ Abgesehen vom Quellterm ist dies ein reines Messgerät.

    3. Massless case $ k ^ 2 = 0 $ und $ k ^ {\ mu} \ neq 0 $. Die Matrix $ M ^ {\ mu} {} _ {\ nu} $ ist nicht diagonalisierbar. Es gibt nur den Eigenwert $ 0 $ (mit der Multiplizität $ n-1 $). Maxwell-Gl. können nur erfüllt werden, wenn die Quelle $ \ tilde {j} ^ {\ mu} (k) = \ tilde {f} (k) k ^ {\ mu} $ proportional zu $ ​​k ^ {\ mu} $ mit ist ein Proportionalitätsfaktor $ \ tilde {f} (k) $. In diesem Fall ist Maxwell Gl. werde $$ -k _ {\ mu} \ tilde {A} ^ {\ mu} (k) ~ = ~ \ tilde {f} (k). $$ Lassen Sie uns einen $ \ eta $ -dualen Vektor $ ^ 1 $ $$ k ^ {\ mu} _ {\ eta} ~: = ~ (-k ^ 0, \ vec {k}) \ qquad {\ einführen rm für} \ qquadk ^ {\ mu} ~ = ~ (k ^ 0, \ vec {k}). $$ Beachten Sie, dass $$ k _ {\ mu} ~ k ^ {\ mu} _ {\ eta} ~ = ~ (k ^ 0) ^ 2 + \ vec {k} ^ 2 $$ ist nur das euklidische Distanzquadrat im Impulsraum $ k ^ {\ mu} $. Die vollständige Lösung ist eine $ (n-1) $ - Parameterlösung der Form $$ \ tilde {A} ^ {\ mu} (k) ~ = ~ - \ frac {k ^ {\ mu} _ {\ eta }} {k _ {\ nu} ~ k ^ {\ nu} _ {\ eta}} \ tilde {f} (k) ~ + ~ ik ^ {\ mu} \ tilde {\ Lambda} (k) ~ + ~ \ tilde {A} ^ {\ mu} _ {T} (k). $$ Der zu $ ​​k _ {\ mu} $ proportionale Term ist reines Maß. Hier bezeichnen $ \ tilde {A} ^ {\ mu} _ {T} (k) $ $ n-2 $ Transversalmodi, $$ k _ {\ mu} ~ \ tilde {A} ^ {\ mu} _ {T. } (k) ~ = ~ 0, \ qquad \ qquadk _ {\ mu} ^ {\ eta} ~ \ tilde {A} ^ {\ mu} _ {T} (k) ~ = ~ 0. $$ Die $ n-2 $ Transversal-Modi $ \ tilde {A} ^ {\ mu} _ {T} $ sind die einzigen sich ausbreitenden physischen d.o.f. (elektromagnetische Wellen, Photonenfeld).

    4. ol>

      -

      $ ^ 1 $ Längs- und zeitliche Polarisationen befinden sich in der masseloser Fall proportional zu $ ​​k ^ {\ mu} \ pm k ^ {\ mu} _ {\ eta} $.

Warum erfüllen die Transversalmodi $ k _ {\ mu} ^ {\ eta} ~ \ tilde {A} ^ {\ mu} _ {T} (k) ~ = ~ 0 $?Maxwell equs.fordere $ k _ {\ mu} ~ \ tilde {A} ^ {\ mu} _ {T} (k) ~ = ~ 0 $.Woher kommt diese zusätzliche Einschränkung?
Es kann als Teil der Definition dessen angesehen werden, was Transversal bedeutet.
1. Aber es gibt keine tatsächliche Einschränkung, die mir sagt, dass sie transversal sein müssen?2. Auch: Was ist das Argument für $ k ^ 2 = 0 $, wenn nicht die Lorenz-Gauge-Maxwell-Gleichung?$ \ Box A ^ \ mu = 0 $?
1. Nein, das gibt es nicht.2. Ich gehe davon aus, dass Sie sich auf Abschnitt III beziehen: Der masselose Fall $ k ^ 2 = 0 $ ist eine Möglichkeit.Es gibt auch einen massiven Fall $ k ^ 2 \ neq 0 $.
Vladimir Kalitvianski
2012-01-27 20:51:16 UTC
view on stackexchange narkive permalink

Gleichungen werden zu jeder Zeit $ t $ geschrieben und es besteht keine Notwendigkeit, ihre Gültigkeit zu irgendeinem Zeitpunkt zu "beweisen". Diese Gleichungen sind die experimentellen Gesetze und natürlich jederzeit konsistent. Die Einschränkungen gelten hier nicht für die Felder, sondern für die elektrischen und magnetischen Ladungen. Die Ladungen haben keine Quellen / Senken, daher sagen die abgeleiteten Gleichungen wie $ \ partiell \ rho / \ partiell t + \ rm div \ vec {j} = 0 $ genau das und werden Ladungserhaltungsgesetze genannt. (Sie sind eine experimentelle Tatsache.) Die Ladungserhaltungsgesetze bestimmen nicht die Ladungsdynamik; für letztere existieren die "mechanischen" Gleichungen. Im Fall einer Elementarladung $ q $ bedeutet ihre Erhaltung ihre Zeitunabhängigkeit: $ \ frac {dq} {dt} = 0 $, die normalerweise nicht als zusätzliche Gleichung geschrieben wird, sondern als Lösung $ q = const $ verwendet wird in den "mechanischen" Gleichungen.

Sie haben also sechs Gleichungen für Felder und zwei als Erhaltungssätze für Ladungen.

Können wir die folgenden zwei Ansichten nicht als äquivalent ansehen: 1. Es gibt sechs Gleichungen für die Felder und es gibt zwei zusätzliche Einschränkungen für die Gebühren, d. H. Die Erhaltungsgesetze.2. Es gibt sechs dynamische Gleichungen für die Entwicklung der Felder und die zwei zusätzlichen Randbedingungen, die zum Anfangszeitpunkt von allen physikalisch realen elektromagnetischen Feldern erfüllt werden müssen.
Shaktyai
2012-07-18 01:52:18 UTC
view on stackexchange narkive permalink

Maxwells Gleichungen sind in der Tat redondant. Wenn man mit den normalen Variablen arbeitet, werden die Redundecies eliminiert. Eine sehr klare Diskussion findet sich in:

Photonen und Atomen: Einführung in die QuantenelektrodynamikClaude Cohen-Tannoudji, Jacques Dupont-Roc, Gilbert Grynberg

Warum ist dies eine Verbesserung der guten Antworten von Lubos und Qmechanic?
Quirino Sugon Jr
2012-02-02 16:10:24 UTC
view on stackexchange narkive permalink

Die Maxwell-Gleichungen bestimmen die elektrischen und magnetischen Felder nicht übermäßig. Dies wird klarer, wenn wir die vier Maxwellschen Gleichungen unter Verwendung der geometrischen Algebra in eine umschreiben: $$ (c ^ {- 1} \ Partial_t + \ vec \ nabla) (\ vec E + i \ zeta \ vec H) = \ zeta (\ rho c + \ vec j) $$, wobei die Vektorprodukte der Pauli-Identität folgen $ \ vec a \ vec b = \ vec a \ cdot \ vec b + i \ vec a \ times \ vec b $. Im Prinzip können wir die Maxwellsche Gleichung invertieren, um das elektromagnetische Feld $ \ vec E + i \ zeta \ vec H $ zu lösen, indem wir Randbedingungen anwenden.

Sie können in TeX schreiben, indem Sie den Texcode mit Dollarzeichen (Inline) oder doppelten Dollars versehen.
user27777
2013-08-03 03:57:45 UTC
view on stackexchange narkive permalink

Hier ist eine verwandte Frage, die ich immer an Schüler stelle. Im freien Raum können Sie die Maxwellsche Gleichung in zwei Helmholtz-Vektorgleichungen umwandeln, eine für E und eine für B. Wie kommt es, dass sie entkoppelt sind? Es scheint, dass wir E und B getrennt berechnen könnten. Der Hinweis ist, dass Sie im freien Speicherplatz einige Randbedingungen angeben müssen, um überhaupt Felder ungleich Null zu haben. Und die Randbedingungen müssen mit den Maxwellschen Gleichungen übereinstimmen. Die transversale, gekoppelte Natur der Felder kommt also vom BC und breitet sich in den freien Raum aus.

Übrigens müssen für endliche Strahlen die E- und B-Felder nicht quer zueinander sein (dh es gibt Längsfelder Felder). Dies macht das Arbeiten mit endlichen Strahlen viel schwieriger als das Arbeiten mit unphysikalischen ebenen Wellen.

auxsvr
2014-04-04 03:58:53 UTC
view on stackexchange narkive permalink

Dies ist leicht zu erkennen, wenn Sie die Maxwell-Gleichungen verwenden, um zu den entkoppelten, inhomogenen Wellengleichungen für die Felder $$ \ begin {split} \ Box \ vec {E} & = - \ mu_0 \ frac {\ zu gelangen teilweise \ vec {J}} {\ teilweise t} - \ vec {\ nabla} \ frac {\ rho} {\ varepsilon_0}, \\\ Box \ vec {B} & = \ mu_0 \ vec {\ nabla} \ mal \ vec {J}, \ end {split} $$ mit $ \ Box \ equiv \ frac {1} {c ^ 2} \ frac {\ partiell ^ 2} {\ partiell t ^ 2} - \ nabla ^ 2 $ der Dalembertianer. Diese Ableitung erfordert die Verwendung aller Maxwell-Gleichungen, und es gibt eine Lösung, die eindeutig definiert ist, wenn wir geeignete Randbedingungen verwenden. Daher sind Maxwells Gleichungen nicht unabhängig.

Ein Hinweis auf ihre Abhängigkeit voneinander ist die Helmholtz-Theorem, vorausgesetzt, die Quellen sind lokalisiert. Gemäß dem Satz ist ein Feld eindeutig definiert, wenn sowohl seine Divergenz als auch seine Krümmung bekannt sind, d. H. Der Satz definiert 3 Funktionen aus 4 abhängigen Gleichungen



Diese Fragen und Antworten wurden automatisch aus der englischen Sprache übersetzt.Der ursprüngliche Inhalt ist auf stackexchange verfügbar. Wir danken ihm für die cc by-sa 3.0-Lizenz, unter der er vertrieben wird.
Loading...